Using Limit Comparison Test to Determine Convergence/Divergence

Click For Summary
The discussion focuses on using the limit comparison test to determine the convergence or divergence of three series. For the first series, the correct comparison series was identified as Bn = 1/n^2, leading to a conclusion of convergence, while Bn = 1/n^3 was deemed inappropriate because it is not larger than An. In the second series, Bn = 1/n^(1/2) was initially chosen, but the correct comparison was 1/n, which is necessary for proper analysis. The third series required a comparison with Bn = 1/n^(5/4) for accurate results, as other choices led to incorrect limits. The key takeaway is that the chosen comparison series must appropriately reflect the behavior of the original series for the limit comparison test to be valid.
izen
Messages
50
Reaction score
0

Homework Statement



Using the limit comparison test to determine convergence or divergence


(1) \sum^{\infty}_{n=1} \frac{(ln(n))^{3}}{n^{3}}


(2) \sum^{\infty}_{n=1} \frac{1}{\sqrt{n} ln(x)}


(3)\sum^{\infty}_{n=1} \frac{(ln(n))^{2}}{n^{\frac{3}{2}}}


Homework Equations





The Attempt at a Solution



I have problem find thing the another series (Bn) compare with An. First I choose Bn when 'n' grows large to represent An

The first question I chose Bn = \frac{1}{n^{3}}
it is the p-series and p>1 so it converges

and then
lim_{n\rightarrow\infty} \frac{\frac{ln(n)^{3}}{n^{3}}}{\frac{1}{n^{3}}}

lim_{n\rightarrow\infty} ln(n)^{3} = \infty

which is wrong so I change to choose Bn = \frac{1}{n^{2}}

lim_{n\rightarrow\infty} \frac{\frac{ln(n)^{3}}{n^{3}}}{\frac{1}{n^{2}}}

lim_{n\rightarrow\infty} \frac{ln(n)^{3}}{n}= 0


So I can conclude that An is convergent by limit comparison

but why I cannot use Bn = \frac{1}{n^{3}} ?

=========================================

(2) and (3) are the same as my first question

(2) \sum^{\infty}_{n=1} \frac{1}{\sqrt{n} ln(x)}

I chose Bn = \frac{1}{n^{1/2}} but after I took the limit of An/Bn. I got 0. In the answer shows that I have to choose Bn = \frac{1}{n^{}}

(3)\sum^{\infty}_{n=1} \frac{(ln(n))^{2}}{n^{\frac{3}{2}}}

I chose Bn = \frac{1}{n^{3/2}} after I took the limit of An/Bn. I got \infty. In the answer shows that I have to choose Bn = \frac{1}{n^{5/4}}

Please give some comments and advices

Thank you
 
Last edited by a moderator:
Physics news on Phys.org
izen said:
but why I cannot use Bn = \frac{1}{n^{3}} ?
The idea is to compare your series with a bigger one with known convergence. 1/n^3 is not bigger, therefore it does not work.

The same happens in the other direction, if you want to show divergence.
 
Thank you for the comments

mfb said:
The idea is to compare your series with a bigger one with known convergence. 1/n^3 is not bigger, therefore it does not work.

The same happens in the other direction, if you want to show divergence.
\sum^{\infty}_{n=1} \frac{(ln(n))^{3}}{n^{3}}

The series (Bn) that we choose must represent the original series (An) and we recognize converges or diverges from Bn. (We don't know from the original). the problem is...

How do I know that \frac{1}{n^3}is too small \frac{(ln(n))^{3}}{n^{3}} and\frac{1}{n^2}not too big for \frac{(ln(n))^{3}}{n^{3}} and why not choose\frac{1}{n}. still not get it.
 
1/n does not give a converging series.
You could choose 1/n3/2 or 1/n5/2 or something similar, if you like, but 1/n2 is easier to study.
 
Question: A clock's minute hand has length 4 and its hour hand has length 3. What is the distance between the tips at the moment when it is increasing most rapidly?(Putnam Exam Question) Answer: Making assumption that both the hands moves at constant angular velocities, the answer is ## \sqrt{7} .## But don't you think this assumption is somewhat doubtful and wrong?

Similar threads

  • · Replies 3 ·
Replies
3
Views
1K
Replies
5
Views
2K
  • · Replies 7 ·
Replies
7
Views
2K
Replies
17
Views
2K
  • · Replies 4 ·
Replies
4
Views
1K
  • · Replies 6 ·
Replies
6
Views
2K
  • · Replies 2 ·
Replies
2
Views
2K
  • · Replies 13 ·
Replies
13
Views
2K
  • · Replies 8 ·
Replies
8
Views
2K
  • · Replies 1 ·
Replies
1
Views
2K